UWorld Gastrointestinal/Nutrition

अब Quizwiz के साथ अपने होमवर्क और परीक्षाओं को एस करें!

Colonscopy - LOWER GI BLEEDING

- involves passing a colonoscope up the rectum into the large intestine to visualize lower GI structures. A colonoscopy is indicated for clients with suspected lower GI bleeding

The nurse assesses for cancer risk factors during a screening event at a gastroenterology clinic. Which of the following client statements include risk factors for esophageal cancer? Select all that apply.

1. "A few years ago, I switched from smoking cigarettes to smoking cigars 1 or 2 times a week." 2."I am proud that I was able to lose 10 lb, but I'm still considered obese for my height." 3."I drink 3 or 4 beers nightly to relax, but I did switch to light beer recently." 4."I have struggled with daily episodes of acid reflux for years, especially at nighttime." Esophageal cancer is a rapidly growing malignancy of the esophageal lining. Risk factors for esophageal cancer include smoking, excessive alcohol consumption, obesity, and gastroesophageal reflux disease. Consumption of salty foods is NOT associated with an increased risk of esophageal cancer but increases the risk of gastric cancer.

Which instructions should the nurse include when providing discharge teaching to a client with peptic ulcer disease due to Helicobacter pylori infection? Select all that apply.

1. "Avoid foods that may cause epigastric distress such as spicy or acidic foods." 2."It is best if you refrain from consuming alcohol products." 3."Report black tarry stools to your health care provider immediately." 4."Take your amoxicillin, clarithromycin, and omeprazole for the next 14 days." Client teaching related to peptic ulcer disease (PUD) includes lifestyle changes (eg, dietary modifications, stress reduction), PUD complications, and medication administration. Helicobacter pylori infection and treatment with nonsteroidal anti-inflammatory drugs (NSAIDs) are risk factors for complicated PUD. H pylori treatment includes antibiotics and proton-pump inhibitors for acid suppression. The recommended initial treatment is 7-14 days of triple-drug therapy with omeprazole (Prilosec), amoxicillin, and clarithromycin (Biaxin). Clients with PUD should avoid NSAIDs [eg, aspirin, ibuprofen (Motrin)] as they inhibit prostaglandin synthesis, increase gastric secretion, and reduce the integrity of the mucosal barrier.

The client has been prescribed a daily dose of oral ferrous sulfate. Which of the following statements should the nurse include in the teaching?

1. "Dark green stools are a harmless side effect of the medication." - Medication adverse effects (eg, dark green stools) that are harmless and do not require intervention (Option 1) 3. "Stool softeners may be necessary to prevent constipation." - Measures to prevent constipation (eg, stool softeners) because iron supplementation can cause adverse gastrointestinal effects (Option 3) 5. "Take ferrous sulfate 1 hour before meals." - Medication administration instructions (eg, taking ferrous sulfate 1 hour before meals) because ferrous sulfate should be taken on an EMPTY stomach because food intake raises gastric pH and impairs iron absorption (OPTION 5) - Iron absorption is ENHANCED not decreased, when it is consumed with vitamin C (eg, orange juice). (Option 4) - The client should avoid taking antacids or calcium supplements within 1 hour of ferrous sulfate because calcium decreases iron absorption.

The nurse is teaching a client with newly diagnosed lactase deficiency about dietary management. Which statements by the client indicate a correct understanding of this condition? Select all that apply.

1. "I can still eat cheese and yogurt as long as they don't make me feel sick." 2."I should take a daily calcium and vitamin D supplement." 4."My lactase enzyme supplement should be taken with meals containing dairy." Clients with lactase deficiency (lactose intolerance) experience varying degrees of gastrointestinal symptoms after ingesting milk products, including flatulence, diarrhea, bloating, and cramping. This is due to a deficiency of the enzyme lactase, which is required for digestion of lactose. Treatment includes restricting lactose-containing foods in the diet. These clients may also take lactase enzyme replacements (eg, Lactaid) to decrease symptoms (Option 4). Supplementation of calcium and vitamin D is recommended due to insufficient intake of fortified milk (Option 2). Milk and ice cream contain the highest amounts of lactose and should be restricted depending on the client's individual tolerance (Option 3). Some dairy products, including aged cheeses and live-culture yogurts, contain little to no lactose and can be tolerated by most clients with lactase deficiency (Option 1).

The nurse is reinforcing teaching about ulcer prevention with a client newly diagnosed with peptic ulcer disease. Which of the following client statements indicate appropriate understanding of teaching? Select all that apply. Peptic ulcer disease (PUD) is characterized by ulceration of the protective layers (ie, mucosa) of the esophagus, stomach, and/or duodenum. Mucosal "breaks" allow digestive enzymes and stomach acid to digest underlying tissues, leading to potential gastrointestinal bleeding and perforation.

1. "I need to avoid taking medicines like ibuprofen without a prescription." 2."I should avoid drinking excess coffee or cola." 3."I should enroll in a smoking cessation program." 4."I should reduce or eliminate my intake of alcoholic beverages." To reduce ulcer formation risk, clients with PUD should be instructed to stop smoking; avoid chronic NSAID use; avoid meals or snacks before sleeping; and limit alcohol and caffeine (cola, tea, coffee) consumption.

The nurse is providing discharge teaching to a client newly diagnosed with ulcerative colitis. Which of the following statements by the client indicate that teaching has been effective? Select all that apply.

1. "I need to eat a diet high in calories and protein so that I avoid losing weight." 2."I need to take multivitamins containing calcium daily." 3."I should avoid consuming alcoholic beverages." 4."I should drink at least 2 liters of water daily and more when I have diarrhea. "5."I will keep a symptom journal to note what I eat and drink during the day." Clients with UC should maintain a high-calorie, high-protein diet (prevent weight loss and muscle wasting); drink at least 2 liters of water per day; take multivitamins as prescribed (i,e, calicum to supplement nutrition); maintain a symptom journal in relation to daily dietary intake; and avoid triggers (caffeine, alcohol, tobacco).

Which statements made by the client demonstrate a correct understanding of the home care of an ascending colostomy? Select all that apply. A colostomy is a surgical procedure that creates an opening (ie, stoma) in the abdominal wall for the passage of stool to bypass an obstructed or diseased portion of the colon. Stool becomes more solid as it passes through the colon

1. "I will clarify with my health care provider before taking enteric-coated medications. 3. "I will limit eating foods such as broccoli and cauliflower to reduce odor." Proper home care of an ascending colostomy includes: - Clarifying enteric-coated medications with a health care provider because these medications may not be properly absorbed in the gastrointestinal tract (Option 1). - Identifying foods that cause excess gas and odor (eg, broccoli, cauliflower, dried beans) (Option 3). Stool produced in the ASCENDING and TRANSVERE colon is semiliquid, which eliminates the need for irrigation. Irrigation to promote a bowel regimen may be useful for descending or sigmoid colostomies because the stool is more solid. Because part of the colon is no longer absorbing water, increased fluid intake should be encouraged. Ensuring sufficient fluid intake (ie, at least 3,000 mL/day unless contraindicated) is necessary to prevent dehydration. The client should empty the pouch when it becomes one-third full to prevent leaks due to increasing pouch weight. Waiting to empty the bag until it is full increases the risk for leaks.

A client is 1-day postoperative abdominoplasty and is discharged to go home with a Jackson-Pratt (JP) closed-wound system drain in place. The nurse teaches the client how to care for the drain and empty the collection bulb. Which statement indicates that the client needs further instruction?

1. "I'll empty the JP bulb when it is totally full so that I don't have to unplug it so many times." - Perform hand hygiene as asepsis must be maintained to prevent the transmission of microorganisms even though there is less chance of bacteria entering the wound using a closed-wound drainage device (eg, JP, Hemovac) than an open-drain device (eg, Penrose) - Pull the plug on the bulb to open the device and pour the drainage into a small, calibrated container (eg, plastic water cup, urine specimen container) as this facilitates recording accurate drainage output (Option 2) - Empty the device every 4-12 hours unless it is 1/2 to 2/3 full before then because as the small capacity bulb (100 mL) fills, the amount of negative pressure in the bulb decreases (Option 1) - Compress the empty bulb by squeezing it from side-to-side with 1 or 2 hands until it is totally collapsed. Although the reservoir can be collapsed by pressing the bottom towards the top, compressing the sides of the reservoir (bulb) is recommended as it is more effective in establishing negative pressure (Option 3) - Clean the spout on the bulb with alcohol and replace the plug when it is totally collapsed to restore negative pressure (Option 4)

A healthy 50-year-old client asks the nurse, "What must I do in preparation for my screening colonoscopy?" Which statements by the nurse correctly answer the client's question? Select all that apply.

1. "No food or drink is allowed 8 hours prior to the test." 4. "The day prior to the procedure your diet will be clear liquids." 5. "You will drink polyethylene glycol as directed the day before." - Clear liquid diet the day before - Nothing by mouth 8-12 hours prior to the examination - The health care provider prescribes a bowel-cleansing agent such as a cathartic, enema, or polyethylene glycol (GoLYTELY) the day before the test. The type of prep depends on the health care provider's preference and client health status. Smoking cessation per se has no role in colonoscopy, but it is good for general health.

During the admission assessment of a client with a small-bowel obstruction, the nurse anticipates which clinical manifestations?

1. Abdominal distention 3. Colicky abdominal pain 4. Frequent vomiting When a small-bowel obstruction develops, fluid and gas collect proximal to the obstruction, producing rapid onset of nausea and vomiting (Option 4), colicky intermittent abdominal pain (Option 3), and abdominal distension (Option 1). Nursing management of an obstruction includes placing the client on NPO status, inserting a nasogastric tube, administering prescribed IV fluids, and instituting pain control measures. Pain during defecation = rectal problem (inflammation, anal fissue, hemorrhoids) Lower large-bowel obstruction = gradual onset of symptoms, cramping abdominal pain, abdominal distension, absolute constipation, and lack of flatus.

he nurse who is caring for a client with acute diverticulitis will immediately report which finding to the health care provider? The normal number of WBCs in the blood is 4,500 to 11,000 WBCs per microliter (4.5 to 11.0 × 109/L). N Diverticulitis (LLQ)

1. Abdominal pain has progressed to the left upper quadrant When these diverticula become inflamed (diverticulitis), the client may experience acute pain (usually in the left lower quadrant/LLQ) and systemic signs of infection (eg, fever, tachycardia, nausea, leukocytosis). Complications that can occur in some clients are abscess formation (continuous fever despite antibiotics and palpable mass) and intestinal perforation resulting in diffuse peritonitis (progressive pain in other quadrants of the abdomen, rigidity, guarding, rebound tenderness). Peritonitis is a potentially lethal complication and should be reported immediately. - Clients with acute diverticulitis can bleed. Usually this bleeding is quite obvious, often with a large amount of bright red blood seen in the stool. This client's mild anemia (11.2) is nonspecific and should not be given reporting priority over the peritoneal signs. -Lying on side with knees drawn up to abdomen and trunk flexed indicates fetal position and could be due to pain. Clients with peritonitis are expected too lie still and take shallow breaths to avoid stretching the inflamed peritoneum/any movement worsens the pain. Peritonitis takes priority over the expected pain in diverticulitis. Leukocytosis is expected with acute diverticulitis. However this client's white blood cell count is only minimally elevated at 12,000 (upper limit of normal is 11,000/mm3 [11.0 x 109/L]) and is not a priority over possible peritoneal signs.

The post-anesthesia care unit nurse receives report on a client after abdominal surgery. What sounds would the nurse expect to hear when auscultating the bowel?

1. Absent bowel sounds Procedures that require bowel manipulation cause a temporary halting of peristalsis (paralytic ileus) for the first 24-48 hours, resulting in absent bowel sounds (Option 1). For bowel sounds to be considered absent, the nurse must auscultate for 2-5 minutes in each quadrant. Peristalsis will usually return in the small intestine in 24 hours, but the large intestine may be delayed 3-5 days Any disease process that causes an increase in peristalsis may cause borborygmi (loud, gurgling sounds). i.e. gastroenteritis, diarrhea, and mechanical obstruction Swishing, buzzing sounds (bruit) and humming sounds heard best with the bell of the stethoscope may be indicative of turbulent blood flow --> aneurysm (arterial dilation) or obstruction (narrowing).

The client is most likely experiencing _____ as evidenced by the client's _____ level.

1. Acute pancreatitis 2. Lipase Lipase is an enzyme that is primarily synthesized in the pancreas to help break down dietary triglycerides into components usable by the body. In acute pancreatitis, serum lipase levels can rise to >3 times the normal upper limit. Therefore, the nurse should suspect that the client is most likely experiencing acute pancreatitis as evidenced by the client's lipase level and associated symptoms (eg, pain after eating, alcohol use, vomiting).

The nurse is reinforcing discharge instructions with a client following a partial gastrectomy. Which of the following instructions should the nurse include to prevent dumping syndrome? Select all that apply. Following a partial gastrectomy, many clients experience dumping syndrome, which occurs when gastric contents empty too rapidly into the duodenum, causing a fluid shift into the small intestine. This results in hypotension, abdominal pain, nausea/vomiting, dizziness, generalized sweating, and tachycardia. The symptoms usually diminish over time.

1. Add high-protein foods to diet 3. Eat small, frequent meals 5. Lie down after eating Recommendations to delay gastric emptying include: - Consume meals high in fat, protein, and fiber, which take more time to digest and remain in the stomach longer than carbohydrates (Option 1). These foods also help meet the body's energy needs. - Avoid consuming fluids with meals because this causes stomach contents to pass faster into the jejunum, which worsens symptoms. Fluid intake should occur up to 30 minutes before or after meals. - Slowly consume small, frequent meals to reduce the amount of food in the stomach (Option 3). - Avoid meals high in simple carbohydrates (eg, sugar, syrup) because these may trigger symptoms when the carbohydrates break down into simple sugars. - Avoid sitting up after a meal because gravity increases gastric emptying. Instead, lying down after meals is encouraged (Option 5).

The nurse has received the following prescriptions. Select the 1 prescription that the nurse should perform first.

1. Administer 1 L lactated Ringer solution IV bolus For a client with acute pancreatitis, aggressive fluid replacement with a lactated Ringer solution IV bolus will maintain adequate circulatory fluid volume, and prevent hypovolemic shock and complications related to electrolyte imbalance (ie, cardiac arrhythmias, neurological complications, acute respiratory distress syndrome, hypotension)

The nurse cares for a client with intractable nausea and vomiting after a total colectomy who has a new prescription for total parenteral nutrition (TPN). Which of the following nursing interventions are appropriate? Select all that apply.

1. Administer TPN through a central venous access device 2.Ensure that dextrose 10% in water IV solution is available 3.Obtain a prescription for blood glucose checks every 4-6 hours 5.Weigh daily with strict documentation of intake and output - Infuse TPN via central venous access devices only , as the concentrated solution may cause phlebitis (inflammation of a vein near the surface of the skin) if given through a peripheral vein (Option 1). Do not piggyback or add medication to TPN infusions. - Do NOT STOP TPN abruptly due to its high concentration of dextrose. If TPN is disrupted, dextrose 10% in water is administered at the prescribed rate of the TPN to prevent hypoglycemia (Option 2). - Check blood glucose every 4 to 6 hours to maintain appropriate glucose levels (Option 3). - Record daily weights and intake and output to ensure the client is receiving adequate nutrition and not developing fluid volume overload (Option 5). A loading dose is NOT given because TPN formulas are uniquely calculated based on the client's individual nutritional demands. Unlike enteral feedings, which are initiated slowly and advanced to a prescribed rate, TPN should ALWAYS be administered at the prescribed rate.

The nurse cares for a client with ulcerative colitis who is having abdominal pain and ≥10 bloody stools per day. Which of the following interventions should be included in the client's plan of care?

1. Administer prescribed analgesic medications as needed 2.Encourage the client to discuss feelings about illness 3.Initiate strict, hourly intake and output monitoring 4.Investigate the client's compliance with the medication regimen 5.Offer the client high-protein foods during meals and snacks - Manage pain: Intestinal inflammation often produces severe abdominal pain that limits treatment compliance. Provide prescribed analgesics to promote comfort and treatment adherence (Option 1). - Address psychosocial needs: Chronic illness may increase the risk of hopelessness and/or depression due to prolonged treatment and frustration over lack of improvement or symptom control. Encourage clients to discuss emotions and feelings (Option 2). - Assess fluid balance: Diarrhea, blood loss, and poor oral intake contribute to dehydration. Strict intake and output monitoring helps ensure adequate fluid intake and prevent dehydration (Option 3). - Evaluate treatment adherence: UC exacerbations may be spontaneous or may be precipitated by certain foods or lack of adherence to prescribed treatments (eg, medications). Assess compliance with prescribed treatments and provide education as needed to promote adherence (Option 4). - Promote nutrition: Pain after eating may lead to anorexia, and intestinal inflammation decreases nutrient absorption; both result in nutritional deficiency. Help clients select nutrient-dense, high-protein, high-calorie foods to promote recovery and meet nutritional needs (Option 5).

The nurse is caring for a client who had an open appendectomy 24 hours ago. Based on the client's clinical data, which of the health care provider's new prescriptions should the nurse clarify? Click the exhibit button for additional information.

1. Advance client to regular diet Postoperative ileus refers to the temporary deceleration of gastrointestinal motility following surgery, which can occur as a result of decreased PO intake, anesthesia, intestinal handling during surgery, and opioid use. Signs of postoperative ileus include abdominal distension, vomiting, and an inability to pass flatus or stool. Following abdominal surgery (eg, appendectomy), clients usually begin a clear liquid diet, and the diet is advanced gradually as gut motility returns to normal (eg, client tolerating intake, passage of flatus, bowel movement, return of bowel sounds). Advancing to a regular diet before the return of peristalsis could cause a bowel obstruction (Option 1). The nurse should assess a hypotensive and tachycardic client for postoperative complications (eg, dehydration, bleeding, sepsis). Despite adequate urine output, additional IV fluids may be indicated to stabilize vital signs.

The nurse is caring for a client who had an endoscopic procedure yesterday to stop upper gastrointestinal bleeding and who started a clear liquid diet today. Which of the following foods would be appropriate to offer to this client? Select all that apply.

1. Apple juice 2.. Chicken broth 5. Unsweatened tea Clear liquids should be introduced slowly and in small amounts to minimize the risk of further gastrointestinal irritation, nausea, and vomiting. Fruit juices with red coloring (eg, cranberry, pomegranate) should not be given to clients with recent gastrointestinal bleeding. If a client vomits, the vomitus may appear red and falsely lead the nurse to believe that the client is bleeding. Red dye-containing foods (eg, red gelatin, cherry popsicles) should also be avoided. Cream of chicken soup and ice cream = full liquid diet

The nurse is caring for a client in the postanesthesia care unit following a gastroduodenostomy (Billroth I). Which of the following nursing interventions are appropriate? Select all that apply. A gastroduodenostomy (Billroth I) involves removing the distal two-thirds of the stomach with anastomosis of the remaining stomach to the duodenum.

1. Applying bilateral sequential compression devices 2.Encouraging splinting of the incision with a pillow when coughing 3.Keeping the client NPO until bowel sounds return Postoperative care of a client with gastroduodenostomy includes initiation of thromboembolism prophylaxis; turning, coughing, and deep breathing; and aspiration precautions (eg, elevating the head of the bed). The nurse should keep clients NPO until bowel sounds return and should not manipulate clogged nasogastric tubes - report to the surgeon.

The nurse is teaching the home health client how to perform colostomy irrigation. Which client action reveals that further teaching is required

1. Attaches an enema set to the irrigation bag, lubricates it, gently inserts it into the stoma, and holds it in place Option 1) A cone-tip applicator is used to instill the irrigation solution into the stoma. An enema set should never be used to irrigate a colostomy. A cone-tip applicator is specifically made to avoid damage to the sensitive colostomy opening. Colostomy irrigation allows the client to create a bowel regimen and to apply a dressing or smaller pouch device over the stoma. To properly irrigate the stoma, use 500-1000 mL of lukewarm water, hang the bag 18-24 inches above the stoma, use the cone-tipped irrigator to slowly infuse the solution, and allow stool to drain through the sleeve into the toilet, but Clamp the tubing if cramping occurs, until it subsides (

The client underwent emergency cholecystectomy and is recovering in the medical-surgical unit. Click to highlight below the findings that are concerning.

1. Bilateral lower lungs have fine crackles that do not clear with coughing. - This may indicate atelectasis, pulmonary edema, or pneumonia. 2. Client has performed incentive spirometry twice since surgery - Incentive spirometry should be performed frequently (eg, 5-10 breaths every hour while awake) to improve ventilation and oxygenation and lower the risk for atelectasis or postoperative pneumonia. Frequent turning, coughing, and deep breathing exercises should also be performed. - Bright red nasogastric (NG) tube drainage: This may indicate active bleeding. - Intolerance of oral intake and no flatus or stool for an extended time: This may indicate postoperative paralytic ileus. Early mobilization and multimodal pain control interventions (eg, cold packs, distraction, nonopioid analgesic medications) help prevent postoperative ileus.

The nurse is caring for a client with liver cirrhosis. Which of the following assessment findings would warrant immediate follow up?

1. Black, tarry stool Black, tarry stools (melena) are caused by digested blood and indicate active upper gastrointestinal (GI) bleeding or bleeding esophageal varices, a potentially life-threatening complication of cirrhosis (Option 1). - Bright red, blood-streaked stool = hemorrhoids or rectal bleeding. Further evaluation of hemorrhoids can be safely delayed. - Biliary tract obstruction (eg, cholelithiasis) = a light gray "clay-colored" - Small, dry, hard stool indicates constipation d/t Inactivity, decreased peristalsis, inadequate fiber intake, decreased fluid intake, and some medications (eg, anticholinergics, opioid analgesics)

Which of the following findings are concerning for malabsorption? Select all that apply. Malabsorption is a condition that occurs when the gastrointestinal (GI) tract is unable to absorb nutrients from food. Common causes include infection, medications (eg, antibiotics), and certain diseases (eg, celiac disease, cystic fibrosis, Crohn disease).

1. Bloating - GI disturbances (eg, abdominal pain, bloating, diarrhea) (Option 1) 2. BMI - Unexplained weight loss (eg, 10 lb [4.5 kg], low BMI [<18.5 kg/m2]) that occurs due to malabsorption of fat and protein 4. Mucosal pallor -Symptoms of anemia due to iron deficiency (eg, pallor, brittle nails) (Option 4) 5. Stool color - Steatorrhea (eg, pale stool color; foul-smelling, oily stool)

The nurse understands that which of these body substances are modes of transmission for hepatitis B?

1. Blood 3. Semen 5. Vaginal secretions One of the most common viral strains that causes hepatitis is hepatitis B. The transmission of hepatitis B is primarily through contact with blood, semen, and vaginal secretions (mnemonic: B for body fluids), commonly through unprotected sexual intercourse and intravenous illicit drug use/sharing needles (Options 1, 3, and 5) Hepatitis A = fecal-oral route via poor hand hygiene and improper food handling.

Diverticulosis = LOWER GI TRACT/BLEEDING

1. Blood vessels surrounding the diverticula weaken the intestinal wall and increase the risk for GI bleeding. 2. Asymptomatic diverticulosis is quite common in elderly people, as in this client.

A nurse is precepting a new graduate nurse who is caring for a client with a paralytic ileus and a Salem sump tube attached to continuous suction. The preceptor should intervene when the graduate nurse performs which interventions? Select all that apply.

1. Checks for residual every 4 hours - Checking for residual volume is not an appropriate intervention because the Salem sump is attached to continuous suction for decompression and is not being used to administer enteral feeding (Option 1). 3. Plugs the air vent if gastric content refluxes - The air vent (blue pigtail) must remain open as it provides a continuous flow of atmospheric air through the drainage tube at its distal end (to prevent excessive suction force). This prevents damage to the gastric mucosa. If gastric content refluxes, 10-20 mL of air can be injected into the air vent. However, the air vent is kept above the level of the client's stomach to prevent reflux (Option 3). General interventions to maintain gastric suction when using a Salem sump tube include: - Maintaining client in semi-Fowler's position - Accurate assessment of bowel sounds (Turn off suction briefly during auscultation) - Keeping the air vent (blue pigtail) open and above the level of the client's stomach - Providing mouth care every 4 hours to maintain moisture of oral mucosa and promote comfort - Inspecting the drainage system for patency ((eg, tubing kink or blockage).

The nurse is reviewing lifestyle and nutritional strategies to help reduce symptoms in a client with newly diagnosed gastroesophageal reflux disease. Which strategies should the nurse include? Select all that apply. Gastroesophageal reflux disease (GERD) occurs when chronic reflux of stomach contents causes inflammation of the esophageal mucosa. The lower esophageal sphincter (LES) normally prevents stomach contents from entering the esophagus. Any factor that decreases the tone of the LES (eg, caffeine, alcohol), delays gastric emptying (eg, fatty foods), or increases gastric pressure (eg, large meals) can precipitate GERD.

1. Choose foods that are low in fat 4. Limit or eliminate the use of alcohol and tobacco 5. Try to avoid caffeine, chocolate, and peppermint Lifestyle and dietary measures that help prevent or minimize symptoms of gastroesophageal reflux disease include avoiding dietary triggers such as alcohol, caffeine, chocolate, peppermint, and high-fat foods. Clients should consume small, frequent meals and discontinue the use of tobacco products.

Select the 3 findings that are most concerning at this time.

1. Coarse crackles auscultated - Clients with acute pancreatitis can develop respiratory complications, including pleural effusions, atelectasis, and acute respiratory distress syndrome. Manifestations of respiratory distress (eg, coarse crackles, tachypnea) should be addressed immediately (Option 2). 2. Hyperglycemia - A blood glucose level that is trending up indicates that the client's pancreas is not releasing enough insulin to maintain appropriate blood glucose levels. Without intervention, the client's blood glucose may continue to trend upward and cause complications secondary to hyperglycemia (eg, dehydration, infection) 3. Temperature 100.8 F (38.2 C) - If pancreatic tissue is severely inflamed or compromised, it may die and become necrotic. Necrotic pancreatic tissue can become easily infected, leading to pancreatic abscess or sepsis. Manifestations of infection (eg, fever) should be reported to the health care provider immediately (Option 9). In addition, this client's pancreatitis may obstruct secretion of both bile and pancreatic lipases, contributing to the development of greasy, clay-colored stools.

A client with diabetes receiving peritoneal dialysis experiences chills and abdominal discomfort. The nurse assesses the client's abdomen by pressing one hand firmly into the abdominal wall. The client experiences pain when the nurse quickly withdraws the hand. The client's most recent blood glucose level is 210 mg/dL (11.65 mmol/L). What is the priority action by the nurse? - Peritonitis is a common but serious complication of peritoneal dialysis that typically occurs as a result of contamination during infusion connections or disconnections. = To detect rebound tenderness, one hand is pressed firmly into the abdominal wall and quickly withdrawn. Rebound tenderness is present when there is pain on removal = indicating inflammation of the peritoneal cavity.

1. Collect peritoneal fluid for culture and sensitivity Typically, the earliest indication of peritonitis = presence of cloudy peritoneal effluent. Later manifestations include low-grade fever, chills, generalized abdominal pain, and rebound tenderness. The nurse should collect peritoneal effluent from the drainage bag for culture and sensitivity (Option 1). Treatment of peritonitis is antibiotic therapy based on the culture results. Antibiotics may be added to dialysate, given orally, or administered intravenously.

The nurse is caring for a client recently diagnosed with a hiatal hernia. Which of the following actions should the nurse take? Select all that apply. Conditions that increase intraabdominal pressure (IAP) (eg, pregnancy, obesity, ascites, tumors) and weaken the muscles of the diaphragm may allow a portion of the stomach to herniate through an opening in the diaphragm, causing a hiatal hernia (upper part of the stomach pushes through/protrudes up an opening in the diaphragm and into the chest cavity) Although hiatal hernias can be ASYMPTOMATIC many clients experience signs and symptoms of gastroesophageal reflux disease (GERD), including heartburn, dysphagia, and pain. To help clients reduce the risk for herniation and associated symptoms, the nurse should instruct the client to:

1. Elevate the head of the bed at least 30 degrees 3.Instruct the client to avoid caffeinated beverages 4.Offer small, frequent, low-fat meals 5.Tell the client to avoid lifting and straining Hiatal hernias develop due to a weakening diaphragm and increased intraabdominal pressure. To reduce herniation, clients should be taught to elevate the head of the bed, avoid high-fat foods and those that relax the lower esophageal sphincter (chocolate, peppermint, tomatoes, caffeine), avoid lifting/straining (ACTIONS THAT INCREASE IAP), and wear loose/nonrestrictive clothing, as well as quit smoking and reduce weight, if appropriate

The nurse should immediately prepare the client for _________ and request a prescription for _________

1. Emergency surgery 2. 0.9% sodium chloride bolus .The nurse should immediately prepare the client for emergency surgery (eg, cholecystectomy) and request a prescription for 0.9% sodium chloride IV bolus to treat severe hypotension (89/58) Peritonitis is a life-threatening inflammation of the peritoneum that occurs when bacteria is introduced into the sterile peritoneal cavity. Manifestations of peritonitis include severe abdominal pain, rigid or "board-like" abdomen, and signs of an inflammatory response (eg, fever, leukocytosis) Endotracheal intubation is indicated for clients requiring support with oxygenation or ventilation. Midazolam IV is a sedative used to facilitate intubation.

The nurse should recognize the client is most at risk for ____ due to ____

1. Gallbladder perforation 2. Biliary obstruction Acute cholecystitis can be caused by gallstones or other mechanical biliary obstruction that blocks bile secretion and causes gallbladder distension. Excessively retained bile may irritate and weaken the gallbladder wall, which can lead to gangrene and gallbladder perforation that can cause peritonitis (ie, life-threatening inflammation and infection of the peritoneum)

The nurse is teaching about the importance of dietary fiber at a community health fair. Which health benefits of consuming a fiber-rich diet should the nurse include in the teaching plan? Select all that apply.

1. Helps prevent colorectal cancer 2. Improves glycemic control 3. Promotes weight loss 4. Reduces risk of vascular disease 5. Regulates bowel movements Dietary fiber (retains water) increases stool bulk and makes stool softer and easier to pass. A fiber-rich diet (fruits, veggies, legumes, and whole grains) helps prevent constipation/improves stool elimination; decreases risk of colorectal cancer; promotes weight loss; (increased satiety as fiber absorbs water and produces fullness) improves blood glucose control; and decreases serum cholesterol levels, which reduces the risk of coronary artery disease and stroke. Fiber binds to cholesterol --> reduces serum cholesterol levels by decreasing the amount of cholesterol that enters blood stream --> reduce vascular plaque buildup and atherosclerosis.

A nurse is caring for a client who developed paralytic ileus (pseudo-obstruction) after a stroke. The client reports nausea, abdominal discomfort, and distension; bowel sounds are absent. Which prescription does the nurse question? Paralytic ileus occurs when the muscle contractions that move food through your intestines are temporarily paralyzed/bowel motor activity is impaired. Sgns and symptoms include abdominal discomfort, distension, and nausea/vomiting. Risk factors for paralytic ileus include: - Abdominal surgery - Perioperative medications (eg, anesthesia, analgesics) - Immobility (eg, stroke)

1. Hydrocodone 5/325 mg 1 tab every 4 hours PRN for moderate pain The client should not take medications by mouth (due to NPO status), and opioid medications should be avoided as they prolong paralytic ileus. Instead, non-opioid IV analgesics (eg, ketorolac, ibuprofen, acetaminophen) should be administered as prescribed if the client is in pain. To prevent further abdominal distension and resulting nausea, the client should remain NPO. Nasogastric tube to wall suction may be necessary to decompress the stomach (Option 3). IV fluid and electrolyte replacement (eg, normal saline) may be necessary to correct losses that occur from nasogastric suction (Option 2). Nausea can be treated with prescribed antiemetics (eg, ondansetron, promethazine) (Option 4).

The nurse suspects the client is experiencing celiac disease and should expect

1. Loss of intestinal villi and mucosal atrophy on small bowel biopsy Diagnosis of celiac disease is typically determined with upper endoscopy and small bowel biopsy, which demonstrate a loss of small bowel intestinal villi and mucosal atrophy. Positive findings for changes in stool form and frequency on the Rome IV criteria are characteristic of irritable bowel syndrome. Skip lesions throughout the gastrointestinal tract on endoscopy and colonoscopy are characteristic of Crohn disease, an inflammatory bowel disease.

During assessment of a client who had major abdominal surgery a week ago, the nurse notes that the incision has dehisced and evisceration has occurred. The nurse stays with the client while another staff member gets sterile gauze and saline. How should the nurse position the client while waiting to cover the wound? Wound evisceration is the protrusion of internal organs through the wall of an incision. It typically occurs 6-8 days after surgery and is more common in clients who have had abdominal surgery, those with poor wound healing, and those who are obese. It is considered a MEDICAL EMERGENCY . The nurse should remain with the client while calling for help. The health care provider should be notified immediately and supplies brought to the room by another staff member.

1. Low Fowler's position with knees bent The wound should be covered with sterile normal saline dressings. While the nurse remains in the room, the client should be positioned in low Fowler's position with the knees bent. This position lessens/REDUCE abdominal tension on the suture line/OPEN WOUND and can prevent further evisceration. The client should be prepared for immediate return to surgery.

Which nursing interventions would the nurse implement when caring for a client newly diagnosed with acute, viral hepatitis B? Select all that apply. Hepatitis (inflammation of the liver) is often caused by infection, toxins, or trauma (eg, drug use, viral hepatitis, acute poisoning), resulting in impairment of liver function (eg, bile production, detoxification of blood, metabolism)

1. Offer small, frequent meals to prevent nausea (anorexia is lowest in the morning so promote eating a larger breakfast). Encourage lOW FAT, SMALL FREQUENT MEALS 2.Promote rest periods between periods of activity 4.Teach the client not to share razors or toothbrushes with others 5.Teach the client to abstain from drinking alcohol - Diets high in fat should be avoided as liver bile production, which is needed for fat digestion, may be impaired. Encourage protein and carbohydrate intake to assist with liver healing. Nursing interventions for clients with acute viral hepatitis include the promotion of rest alternated with activity, avoidance of hepatotoxic substances (eg, alcohol), and adequate nutrition (adequate carbohydrates and protein intake; low fat; small, frequent meals).

A homeless man known to have chronic alcoholism and who has not eaten for 8 days is undergoing nutritional rehabilitation via oral and enteral feedings. Which of the following findings would indicate that the client is developing refeeding syndrome?

1. Phosphorus 2.0 mg/dL (0.65 mmol/L), potassium 2.9 mEq/L (2.9 mmol/L), magnesium 1.0 mEq/L (0.5 mmol/L) Refeeding syndrome is a potentially lethal complication of nutritional replenishment in significantly malnourished clients and can occur with oral, enteral, or parenteral feedings. After a period of starvation, carbohydrate-rich nutrition (glucose) stimulates insulin production along with a shift of electrolytes from the blood into tissue cells for anabolism. The key signs of refeeding syndrome are rapid declines in phosphorous, potassium, and/or magnesium (mnemonic PPM). Other findings may include fluid overload, sodium retention, hyperglycemia, and thiamine deficiency. Actions to prevent refeeding syndrome include the following: - Obtaining baseline electrolytes - Initiating nutrition support cautiously with hypocaloric feedings - Closely monitoring electrolytes - Increasing caloric intake gradually

The nurse is caring for the client in the emergency department. Click to highlight below the 3 findings that require immediate follow-up.

1. T 100.4 F (38 C), P 112 2. Pain in the epigastric area and right upper abdominal quadrant that radiates to the right shoulder 3. tenderness with guarding are noted during palpation - Epigastric pain and right upper abdominal pain radiating to the shoulder, jaw, or arm: This may suggest acute cholecystitis (ie, inflammation of the gallbladder), atypical presentation of myocardial infarction (MI), or blood in the peritoneal cavity irritating the diaphragm (eg, ruptured spleen, ectopic pregnancy). - Abdominal tenderness with guarding noted during palpation of the right upper quadrant (RUQ): This is often associated with inflammatory processes of the biliary tract. - Alterations in vital signs: Fever and tachycardia may indicate an infectious process. - Increased pain with deep inspiration during palpation of the RUQ (ie, positive Murphy sign): This is a signature finding associated with acute cholecystitis. - Severe nausea and vomiting: These are common presenting symptoms of MI and several infectious (eg, gastroenteritis), inflammatory (eg, cholecystitis), or obstructive (eg, small bowel obstruction) gastrointestinal disease processes.

The nurse prepares to assess a newly admitted client diagnosed with chronic alcohol abuse whose laboratory report shows a magnesium level of 1.0 mEq/L (0.5 mmol/L). Which assessment finding does the nurse anticipate?

1. Tremors and brisk deep-tendon reflexes Hypomagnesemia, a low blood magnesium level (normal 1.5-2.5 mEq/L [0.75-1.25 mmol/L]), is associated with alcohol abuse due to poor absorption, inadequate nutritional intake, and increased losses via the gastrointestinal and renal systems. It is associated with 2 major issues: Ventricular arrhythmias (torsades de pointes): This is the most serious concern (priority). Neuromuscular excitability: Manifestations of low magnesium, similar to those found in hypocalcemia and demonstrated by neuromuscular excitability, include tremors, hyperactive reflexes, positive Trousseau and Chvostek signs, and seizures. Hypercalcemia: constipation and polyuria (calcium = diuretic effect_ Hyernatremia (sodium) = increased thirst with dry mucous membranes Hypokalemia (potassium) = muscle weakness/paralysis and soft flabby muscles. Common is paralytic ileus (abdominal distention, decreased bowel sounds)

Click to highlight below the 4 assessment findings that are most concerning at this time.

1. epigastric and left upper quadrant abdominal pain that radiates to the back. - may indicate severe cardiovascular conditions (eg, aortic dissection, abdominal aortic aneurysm) and should be assessed immediately. 2. The client's pain worsens after eating or drinking, - Pain that worsens with eating may indicate the presence of a gastric ulcer, which is concerning for possible upper gastrointestinal (GI) bleeding. 3. the client has had multiple episodes of greenish-brown emesis - that increases the likelihood of electrolyte imbalances (eg, hypokalemia, hyponatremia) that could lead to life-threatening cardiac arrhythmias or neurological complications (eg, seizure, coma). 4. consumes alcohol daily - A history of smoking and alcohol consumption, and a family history of peptic ulcer disease increase the client's risk of upper GI bleeding. In the presence of excessive vomiting, weight loss is concerning for malnutrition; however, the nurse should address other client findings prior to nutritional intake.

The health care provider prescribes a small bowel follow-through for a client. Which of the following information should the nurse reinforce to the client about this test?

2. "During the test, a series of x-rays will be taken to assess the function of the small bowel." A small bowel follow-through (SBFT) is performed to examine the anatomy and function of the small intestines using x-ray images taken in succession. Barium is ingested, and x-ray images are taken every 15-60 minutes to visualize the barium as it passes through the small intestine (Option 2). Prior to an SBFT, the client should be NPO for at least 8 hours. The client should drink plenty of fluids after an SBFT to facilitate barium removal. Chalky stools may be present for 24-72 hours after the procedure; however, if brown stools do not return after 72 hours or if abdominal pain/fullness is present, the client should contact the health care provider (HCP). Polyethylene glycol is prescribed as a bowel preparation for a colonoscopy, not an SBFT.

A student nurse asks why enteral (tube) feedings, rather than total parenteral nutrition (TPN), are being administered to a client with sepsis and respiratory failure. Which is the best response by the registered nurse?

2. "Enteral feedings maintain gut integrity and help prevent stress ulcers. The early initiation of enteral feedings helps preserve the function of the gut mucosa, limits movement of bacteria (translocation) from the intestines into the bloodstream, and prevents stress ulcers. Enteral feedings are also associated with lower risk of infectious complications compared with TPN. However, the mortality is the same. Caloric and metabolic needs can usually be met adequately using enteral feedings or TPN.

The nurse provides discharge instructions to a client one day after laparoscopic cholecystectomy. Which statement by the client indicates that further teaching is required? Laparoscopic cholecystectomy is the most common procedure for gallbladder removal. A laparoscope and grasping forceps are inserted through small punctures made in the abdomen. The procedure is associated with decreased postoperative pain, better cosmetic results, shorter hospital stays, and fewer days for recovery compared with the open technique.

2. "I can take a bath as long as I do not scrub the surgical glue off of the incision." Surgical glue is a form of topical adhesive that approximates the edges of a wound. Clients with surgically glued incisions may shower following surgery; however, soaking (eg, bathing, swimming) or scrubbing the incision site may remove the glue, impairing wound healing and increasing the risk of infection (Option 2).

The registered nurse (RN) is supervising a graduate nurse (GN) providing postoperative teaching for a male client after an inguinal hernia repair. Which statement by the GN would cause the RN to intervene? An inguinal hernia is the protrusion of abdominal contents through the inguinal canal, which appears as a bulge in the lateral groin. Herniation occurs spontaneously or results from increased intraabdominal pressure (eg, heavy lifting

2. "Practice coughing to clear secretions and prevent pneumonia To prevent hernia reoccurrence after surgical repair, the client is taught to avoid activities that increase intraabdominal pressure (eg, coughing, heavy lifting) for 6-8 weeks (Option 2). If sneezing or coughing are unavoidable, the client should splint incisions and keep the mouth open while sneezing. After inguinal hernia repair surgery, clients should avoid coughing and heavy lifting, ambulate early, turn and deep breathe every 2 hours, and stand when voiding. Scrotal elevation and ice packs help decrease pain and swelling.

A client with end-stage liver disease is admitted for a transplant workup. The client's spouse states that the client has not stopped drinking alcohol and may be unable to quit for 6 months before the transplant. Which is the most appropriate action for the nurse to implement?

2. Assess the client's motivation to make the necessary self-care changes before and after the transplant The client may not be an appropriate transplant candidate due to his alcohol use. However, additional facts are needed to determine the true situation as the only information obtained came from the client's spouse. The nurse should assess the client's drinking habits and motivation to stop drinking before and after the transplant by speaking with the client directly. In addition, a transplant requires many other self-care regimens. The nurse should be alert for indicators of the client's ability to take prescribed medications, follow dietary restrictions, and attend medical appointments. The information obtained from this assessment should be communicated to the interdisciplinary team members responsible for determining transplant eligibility.

The nurse is providing postoperative care to a client who had a laparoscopic cholecystectomy 4 hours ago. Which intervention is the priority? In laparoscopic cholecystectomy, carbon dioxide (CO2) is used to inflate and expand the abdominal cavity to allow insertion of surgical instruments and better visualization of organs. CO2 can irritate the phrenic nerve and diaphragm, causing shallow breathing and referred pain to the shoulder.

2. Assist with early ambulation The nurse should assist the client with early ambulation and deep breathing to facilitate dissipation of the CO2 (Option 2). Early ambulation improves breathing, decreases the risk for thromboembolism, and stimulates bowel motility (ie, peristalsis). Clients who have laparoscopic procedures are often discharged the same or next day, so initiating mobility is a priority. After laparoscopic cholecystectomy, the client should maintain a clear liquid diet until nausea and pain subside, typically advancing the diet after discharge. A low-fat diet is recommended for several weeks after surgery to help prevent bile duct pain.

The nurse is providing nutritional teaching for a client with a new ileostomy. Which foods should the nurse instruct the client to avoid? Select all that apply. An ileostomy is a surgically created opening (stoma) in the abdominal wall that connects the small intestine to the external abdomen. Stool from the small intestine bypasses the colon and exits through the ileostomy. Functions of the colon (eg, fluid and electrolyte absorption, vitamin K production) do not occur, resulting in liquid stool that drains into an external ostomy appliance attached to the skin.

2. Broccoli with cheese 3. Multigrain bagel 4. Popcorn Foods to be avoided include: - High fiber: popcorn, coconut, brown rice, multigrain bread (Options 3 and 4) Stringy vegetables: celery, broccoli, asparagus (Option 2) - Seeds or pits: strawberries, raspberries, olive - Edible peels: apple slices, cucumber, dried fruit In the immediate postoperative period of an ileostomy, a low-residue diet (low-fiber) is prescribed to prevent obstruction of the narrow lumen of the small intestine and stoma (1-in [2.54-cm] diameter or less). After the ileostomy heals, the client reintroduces fibrous foods one at a time. The client is instructed to thoroughly chew food and monitor for changes in stool output. - After an ileostomy, a client may consume fruits and vegetables that are pitted, peeled, and/or cooked (eg, peaches, bananas, potatoes).

A client receiving total parenteral nutrition complains of nausea, abdominal pain, and excessive thirst. What is the best action for the nurse to take?

2. Check the client's blood glucose A complication of total parenteral nutrition (TPN) is hyperglycemia, as evidenced by excessive thirst, increased urination, abdominal pain, headache, fatigue, and blurred vision. The development of hyperglycemia: - Excessive dextrose infusion - A low tolerance for dextrose in critically ill clients due to the inflammatory response and the resulting production of counterregulatory hormones - High infusion rate - Administration of medications such as steroids Infection Interventions to resolve TPN-associated hyperglycemia include reducing the amount of carbohydrate in the TPN solution, slowing down the infusion rate, and administering subcutaneous insulin.

The nurse is caring for a client with a balloon tamponade tube in place due to bleeding esophageal varices. The client suddenly develops respiratory distress, and the nurse finds that the tube has been partially pulled out. Which intervention should be the nurse's priority?

2. Cut the tube with scissors. A balloon tamponade tube (eg, Sengstaken-Blakemore, Minnesota) is used to temporarily control bleeding from esophageal varices. It contains 2 balloons and 3 lumens. The gastric lumen drains stomach contents, the esophageal balloon compresses bleeding varices above the esophageal sphincter, and the gastric balloon compresses from below. A weight is attached to the external end of the tube to provide tension and hold the gastric balloon securely in place below the esophageal sphincter. Airway obstruction can occur if the balloon tamponade tube becomes displaced and a balloon migrates into the oropharynx. Scissors are kept at the bedside as a precaution; in the event of airway obstruction, the nurse can emergently cut the tube for rapid balloon deflation and tube removal (Option 2). If airway obstruction occurs, the nurse should first clear the airway and then ensure that the client is stable before contacting the health care provider. Low intermittent suction to the gastric lumen of a balloon tamponade tube is used to drain stomach contents.

The nurse is caring for a client with right upper quadrant pain and jaundice. The client's alanine aminotransferase /aspartate aminotransferase (ALT/AST) levels are 7 times the normal values. What questions would be most helpful regarding the etiology for these findings? Select all that apply.

2. Do you use intravenous (IV) illicit drugs? 3. How much alcohol do you typically drink? 5. What over-the-counter drugs? ALT and AST are the enzymes released when hepatic cells are injured (hepatitis). If you have high levels of AST and/or ALT, it may mean that you have some type of liver damage Besides viral hepatitis, liver injury can occur with excessive chronic alcohol intake (Option 3), some over-the-counter medications (eg, acetaminophen), and certain herbal and dietary supplements (Option 5). IV illicit drug use increases the risk for hepatitis B and C infection (Option 2). - Black tarry stool (melena) = gastrointestinal bleed

The nurse is counseling a client with obesity who is starting a weight reduction diet. The client reports consuming 4-5 regular cola beverages daily. Which of the following beverages should the nurse recommend as healthier substitutes? Select all that apply.

2. Flavored club soda 3.Fresh vegetable juice 5.Unsweetened tea Individuals who are attempting to lose weight should consume beverages with nutritional value and little-to-no caloric value, including: - Water - Club soda (flavored or unflavored) (Option 2) - Unsweetened tea and/or coffee (Option 5) - Fresh vegetable juice (Option 3) - Nonfat or low-fat milk (in limited amounts) Commercially available fruit juices and sports beverages often contain relatively high amounts of sugar and calories with little to no nutritional value (ie, "empty calories"), and should be limited or removed from the client's dietary intake.

A nurse is evaluating the teaching of weight reduction strategies to a client with obesity. Which of the following statements indicate that the client understands the teaching? Select all that apply.

2. I will aim to lose 1-2 lb per week 3. I will keep healthy snacks on hand in case I get hungry 5. I will take the stairs instead of the elevator. To promote effective weight loss, the nurse teaches the client to eliminate sugar-containing beverages; aim to lose about 1-2 lb (0.45-0.91 kg) per week; eat small, frequent meals; incorporate daily exercise with increasing intensity; and get about 7-9 hours of sleep per night.

The hospitalized client with anorexia nervosa is started on central parenteral nutrition. Which client assessment is the most important for the nurse to check during the first 24-48 hours of administration? Refeeding syndrome is a potentially fatal complication of nutritional rehabilitation in chronically malnourished clients (eg, anorexia nervosa, chronic alcoholism). Lack oral intake -> decreased insulin production.

2. Serum potassium and phosphate After the client receives food or IV fluids with glucose, insulin secretion is increased, leading to phosphorous, potassium, and magnesium shifting intracellularly. Hypophosphatemia = muscle weakness and respiratory failure. Deficiencies in potassium and magnesium = cardiac dysrhythmias. Therefore, checking serum potassium, phosphate, and magnesium levels is necessary when implementing aggressive nutrition supplementation (Option 2). Dumping syndrome is seen after gastric surgeries that result in decreased storage area in the stomach. . Symptoms include diaphoresis, cramping, weakness, and diarrhea after eating Central parenteral nutrition is delivered via a central line, which carries a risk for infection. The signs of infection includes leukocytosis and left shift. However, risk of infection is low in the first few days of parenteral nutrition.

The nurse is caring for a client with end-stage liver failure from hepatitis C who is being seen in the clinic for worsening ascites. The client is treated in the infusion center with intravenous (IV) albumin, IV furosemide, and oral spironolactone. The following day the nurse checks the client's labs. Which of the following lab findings is most important for the nurse to communicate to the health care provider? Spironolactone is a potassium-sparing diuretic (water pill). It prevents your body from absorbing too much salt and keeps your potassium levels from getting too low. This medicine is also used to treat or prevent hypokalemia (low potassium levels in the blood A normal albumin range is 3.4 to 5.4 g/dL. If you have a lower albumin level, you may have malnutrition. It can also mean that you have liver disease, kidney disease, or an inflammatory disease. Higher albumin levels may be caused by acute infections, burns, and stress from surgery or a heart attack. Furosemide induces various electrolyte imbalances including hypokalemia, hypomagnesemia, hypocalcemia, hyponatremia, and hyperuricemia3,16). Hypokalemia is caused by the increased distal delivery of potassium and secondary mi

3, Potassium 3.0 mEq/L (3.0 mmol/L) The client with cirrhosis is at risk of hepatic encephalopathy. Hypokalemia, high protein intake, gastrointestinal bleeding, constipation, hypovolemia, and infection can precipitate hepatic encephalopathy. Use of furosemide can cause hypokalemia, which must be corrected immediately to prevent the precipitation of hepatic encephalopathy and dangerous ventricular arrhythmias --> cardiac arrest. Lab abnormalities common in liver failure include low albumin, elevated INR, and elevated liver function tests. A low serum potassium can increase the risk of hepatic encephalopathy and should be reported to the health care provider. Elevated serum ammonia confirms the hepatic encephalopathy diagnosis.

An adult diagnosed with celiac disease 3 weeks ago was placed on a gluten-free diet. The client returns for ambulatory care follow-up, reports continuation of symptoms, and does not seem to be responding to therapy. Which is the best response by the nurse?

3. "Tell me what you had to eat yesterday." This client with celiac disease continues to have symptoms. An assessment of the client's food intake must be obtained to determine if it includes foods that contain gluten, a protein in barley, rye, oats, and wheat (mnemonic: BROW). The most common reason for non-responsiveness to a gluten-free diet in clients with celiac disease is that gluten has not been entirely eliminated from their food intake.

The nurse on the medical-surgical unit receives report on assigned clients. Which client warrants immediate attention? Endoscopic retrograde cholangiopancreatography (ERCP) is a procedure in which an endoscope is passed through the mouth into the duodenum to assess the pancreatic and biliary ducts. Using fluoroscopy with contrast media, the ducts can be visualized and treatments including removal of obstructions, dilation of strictures, and biopsies can be performed.

3. Client with epigastric pain after endoscopic retrograde cholangiopancreatography Perforation or irritation of these areas during the procedure can cause acute pancreatitis (LUQ), a potentially life-threatening complication after an ERCP. Signs and symptoms include acute epigastric or left upper quadrant pain, often radiating to the back, and a rapid rise in pancreatic enzymes (eg, amylase, lipase) (Option 3). - Abdominal cramps can occur after a colonoscopy due to air inflation during the procedure. - (Option 2) The barium contrast solution used during the procedure may make the client's stool white for up to 3 days. The nurse should encourage fluids, if appropriate, to assist in expulsion of the contrast medium. - (Option 4) Copious, bile-colored (greenish-brown) drainage is EXPECTED in a client with a small bowel obstruction. The nurse should watch for signs and symptoms of electrolyte imbalances (hypokalemia), dehydration, and metabolic alkalosis.

The nurse is developing teaching materials for a client diagnosed with ulcerative colitis. The client will receive sulfasalazine. Which of the following instructions are included in the discharge teaching plan? Select all that apply Ulcerative colitis (UC) is a chronic inflammatory bowel disease characterized by inflammation and ulceration of the large intestine (colon) that results in abdominal pain, frequent episodes of bloody diarrhea, anorexia, and anemia. During UC exacerbations, clients have numerous bowel movements (≥6/day), which can lead to fluid loss, malabsorption of dietary nutrients, and perianal skin breakdown.

3. Continue sulfasalazine even after resolution of symptoms 4.Increase fluid intake to at least 2000 mL/day 5.Take daily vitamin and mineral supplements 6.Use witch hazel for perianal irritation For a client recovering from an acute ulcerative colitis exacerbation, discharge teaching should include avoiding triggers (CAFFEINE/coffee, alcohol, fatty foods, excessive seasoning) to decrease risk of cramping and diarrhea, continuing sulfasalazine after symptoms resolve to prevent relapse, drinking at least 2000-3000 mL of fluid daily to maintain hydration and electrolyte balance, taking daily vitamins and supplements, and using witch hazel, zinc cream, or medicated sitz bath for perianal irritation and to prevent skin breakdown.

Which group of food selections would be appropriate for a client on a full liquid diet 3 days after bariatric surgery?

3. Creamy wheat cereal, blended cream of chicken soup, and a protein shake Bariatric surgery REDUCES stomach capacity --> diet: LOW in simple carbohydrates and HIGH in protein, low in sugar too. After gastric surgery, consumption of simple carbohydrates can lead to dumping syndrome (ie, cramping, diarrhea). Small, frequent meals are recommended to avoid overstretching of the gastric pouch and to prevent nausea, vomiting, and regurgitation. The best food choices for a bariatric full liquid diet are cream soups, refined cooked cereal, sugar-free drinks, low-sugar protein shakes, and dairy foods

The nurse teaching a group of clients about celiac disease will include which meal in the teaching plan?

3. Grilled chicken, green beans, and mashed potatoes Celiac disease is an autoimmune disorder in which chronic inflammation caused by gluten damages the small intestine. The following are important dietary principles to teach clients with celiac disease All gluten-containing products should be eliminated from the diet. These include wheat, barley, breads rye, and oats, pasta. Rice, corn, and potatoes are gluten free and are allowed on the diet (Option 3). Processed foods = hidden sources of gluten such as starch, malt, and soy sauce. Gluten free diet for the rest of the lives. Acceptable: baked salmon, rice, and steamed vegetables.

A 78-year-old client recovering from a hip fracture tells the home health nurse, "I haven't had much of an appetite lately and have been really tired. I'm worried I'm not eating enough." Which question is the priority for the nurse to ask?

3. Have you lost any weight unintentionally? Assessing for malnutrition involves collecting dietary data (eg, 24-hour diet recall), laboratory values (eg, albumin or prealbumin), physical measurements (eg, BMI), and history of recent weight loss (Option 3). Reports of weight loss, especially unintentional, are critical findings often indicative of malnutrition. In addition, weight loss of ≥5% in 1 month or ≥10% in 6 months may indicate serious conditions (eg, cancer, tuberculosis, failure to thrive). Before determining the cause of the client not eating enough food, the nurse needs to determine if this is the case or not (i.e. is the client malnourished?). Options 1, 2, 4: These questions attempt to discover why/contributing factors of malnutrition (eg, functional status, mood alteration, diet) the client has inadequate nutritional intake; we want to discover if there even is inadequate intake first.

The nurse provides discharge instructions to a client with cirrhosis who has portal hypertension, ascites, and esophageal varices. Which statement by the client indicates that the teaching was effective? Cirrhosis is a progressive, degenerative disease caused by destruction and subsequent disordered regeneration of the liver parenchyma. Portal HTN: elevated pressure in your portal venous system. The portal vein is a major vein that leads to the liver. Esophageal varices are enlarged veins in the esophagus, the tube that connects the throat and stomach.

3. I should avoid straining while having a bowel movement." Clients with cirrhosis suffer from various complications (eg, ascites, varices, encephalopathy) that will progressively intensify without lifestyle modifications. The nurse should instruct the client to avoid activities that increase intraabdominal pressure (eg, straining, heavy lifting), which could cause rupture of varices (Option 3). Clients with cirrhosis should eat a high-calorie, high-carbohydrate, low-sodium, and low-fat diet; moderate protein intake is recommended. They should avoid hepatotoxic substances (eg, alcohol, acetaminophen) and medications (NSAIDs) that increase bleeding risk and reduce activities that increase intraabdominal pressure.

The nurse admits a client with cirrhosis who has an upper gastrointestinal bleed from suspected gastroesophageal varices. Which new prescription should the nurse question? Upper gastrointestinal bleeding (UGIB) is a potentially life-threatening condition commonly caused by bleeding gastroesophageal varices or peptic ulcers. Gastroesophageal varices are distended, fragile blood vessels within the stomach and/or esophagus that frequently occur secondary to cirrhosis. Due to the fragility of these veins, clients are closely monitored for variceal rupture. Rupture of gastroesophageal varices is an emergency complication that rapidly results in massive gastrointestinal bleeding, hypovolemic shock, and death.

3. Insert and maintain a nasogastric tube Variceal rupture commonly occurs due to a sudden increase in portal venous pressure (eg, coughing, straining, vomiting) and from mechanical injury (eg, chest trauma, consuming sharp/hard foods). In UGIB, nasogastric tube insertion may be prescribed for gastric decompression or evacuation. However, nasogastric tube insertion without visualization of the esophagus may traumatize and rupture varices, causing hemorrhage (Option 3). - Pantoprazole (PPI) is prescribed for clients with UGIB to reduce gastric acid secretion and help prevent ulceration of the gastric mucosa. (Option 2) Octreotide may be used to help control UGIB related to bleeding gastroesophageal varices, as it reduces portal venous pressure, which reduces bleeding. (Option 4) NPO status may be prescribed in cases of UGIB to prepare the client for invasive diagnostic or therapeutic procedures (eg, esophagogastroduodenoscopy, variceal ligation).

During assessment of a client with cholelithiasis (or gallstones are hardened deposits of digestive fluid that can form in your gallbladder) and acute cholecystitis (gallblader), which of the following findings should the nurse expect during the health history and physical examination? Select all that apply. Acute cholecystitis often occurs after a gallstone has obstructed the cystic duct (ie, cholelithiasis), causing the gallbladder to become inflamed, distended, and infected. Potential complications of acute cholecystitis include perforation of the gallbladder, abscess formation, and peritonitis. Clients are often admitted for supportive or surgical management (ie, laparoscopic cholecystectomy).

3. Low grade fever with chills 5. Right upper quadrant (RUQ) pain radiating to the right shoulder. Cardinal symptoms of acute cholecystitis include colicky pain in the right upper quadrant with referred pain to the right shoulder and scapula (Option 5). Clients often report fatty food ingestion 1-3 hours before the initial onset of pain. Fatty foods signal the gallbladder to contract and empty bile to help with digestion in the small intestine. Associated symptoms also include chills, nausea, vomiting, anorexia, and low-grade fever (Option 3). Renal calculi (ie, nephrolithiasis) = Flank pain radiating to the groin would be It is NOT dietary protein but foods with significant fat content (eg, cheese, avocado, fried foods) that signal the gallbladder to contract, emptying bile into the small intestine to help digestion. The initial onset of pain at the umbilicus would be expected in clients with acute appendicitis.

A 70-year-old client is admitted to the hospital with a lower gastrointestinal bleed. After assisting the client back to bed, the nurse finds approximately 600 mL of frank red blood in the commode. The client is now pale and diaphoretic and reports dizziness. Which action should the nurse perform first?

3. Lower the head of the bed Acute blood loss is a medical emergency, and the nurse needs to carry out interventions rapidly. Lowering the head of the bed or placing the client in the supine position maintains blood perfusion to the brain and other vital organs. This can be done quickly to help stabilize the client before performing other intervention

The nurse is caring for a client with cirrhosis who has ascites, peripheral edema, shortness of breath, fatigue, and generalized discomfort. Which of the following actions should the nurse take? Select all that apply.

3. Maintain the client in semi-Fowler position - Maintaining the client in semi-Fowler or Fowler position can promote comfort and reduce pressure on the diaphragm (Option 3). Side-lying with the head of the bed elevated can also promote comfort for the client because it allows the heavy, enlarged abdomen to rest on the bed, reducing pressure on internal organs and allowing for relaxation. 4.Provide an alternating air pressure mattress for the client - Meticulous skin care is a priority due to the increased susceptibility of skin breakdown from edema, ascites, and pruritus. The nurse should provide a specialty mattress (eg, alternating air pressure mattress) and implement a turning schedule of every 2 hours (Option 4). 5.Use music to provide a distraction for the client = Distraction can be used to take the client's mind off the current symptoms and promote comfort. Distractions include listening to music, watching television, or taking part in hobbie

A client calls the primary care clinic reporting diarrhea for 4 days and a low grade fever. What instruction is most important for the nurse to give to the client?

3. Make an appointment for the client with the health care provider today Most bouts of diarrhea are self-limiting and last ≤48 hours. Clients experiencing diarrhea that lasts >48 hours or accompanied by fever or bloody stools should be evaluated by a health care provider (HCP). Causes may include infectious agents, dietary intolerances, malabsorption syndromes, medication side effects, or laxative overuse. The HCP will need to assess for dehydration and electrolyte imbalances and identify underlying causes of the diarrhea that may require further treatment (eg, Clostridium difficile). Instructions on rest, fluids, and acetaminophen are helpful and would be the primary choice if the diarrhea had been occurring ≤48 hours without other symptoms. Loperamide (Imodium) is a synthetic opioid used as an antidiarrheal. It slows peristalsis and subsequently increases fluid absorption. It should not be used more than 2 days or if fever is present as retention of bacteria or toxins inside the colon can make the process worse and cause toxic megacolon.

The nurse is caring for a client who underwent an endoscopic sleeve gastrectomy this morning for the treatment of morbid obesity. Based on the client's clinical data, which action by the nurse is appropriate at this time? Click the exhibit button for more information. Clients with morbid obesity who are unable to achieve weight reduction with conservative treatment (eg, lifestyle modification, medications) may choose to undergo bariatric surgery. Weight reduction from bariatric surgery occurs by restricting the stomach's capacity to hold food and/or reducing food absorption. During sleeve gastrectomy, most of the stomach is removed, and the remaining portions are attached together, creating a "sleeve" or pouch

3. Notify the health care provider or surgeon immediately After bariatric surgery, the nurse must closely monitor for clinical indicators of complications. Severe abdominal pain (especially radiating to the back/shoulder) accompanied by signs of sepsis (eg, restlessness, tachycardia, oliguria) may indicate an anastomotic leak, which occurs when gastric and/or intestinal contents leak through a surgical connection into the intrabdominal space. u(Option 3).

THE graduate nurse (GN) is caring for a client with acute appendicitis (RLQ) who is awaiting surgery. Which action by the GN would require the precepting nurse to intervene?

3. Provides a heating pad for abdominal discomfort Nurses caring for clients with appendicitis should AVOID interventions that increase intestinal blood circulation, gut motility, or appendiceal intraluminal pressure. The application of heat to the abdomen (eg, heating pad, warm blanket) INCREASES INTESTINAL CIRCULATION AND THE RISK FOR APPENDICEAL PERFORATION (Option 3). Food and drink increase gastric motility, thereby increasing circulation to the appendix and risk of perforation. The nurse should teach the client about remaining NPO before surgery.

A client with a 10-year history of unipolar major depression has relapsed and is now hospitalized. The client is currently on phenelzine (Nardil) and weighs 115 lb (52.1 kg) but weighed 150 lb (68 kg) 3 months prior to admission. Which foods would be the best for this client? Phenelzine is a potent, non-selective monoamine oxidase inhibitor (MAOI) used in treating depression and as an anxiolytic in adults Top MAOI: Isocarboxazid (Marplan) Phenelzine (Nardil) Selegiline (Emsam) Tranylcypromine (Parnate) The client is taking phenelzine (Nardil), which is a monoamine oxidase inhibitor (MAOI). Foods high in tyramine (eg, aged cheese, yogurt, cured meats, fermented foods, broad beans, beer, red wine, chocolate, avocados) need to be restricted to reduce the risk of a hypertensive crisis.

3. Steamed fish and potatoes Reduced appetite and significant, unintentional weight loss are included in the diagnostic criteria for unipolar major depression (major depressive disorder). A 35-lb (15.9-kg) weight loss within 3 months is a 23% change in this client's usual body weight and is considered severe weight loss. The client needs a diet high in calories AND protein to promote adequate nutrition and weight gain. In addition, the client has a diagnosis of depression and may have a low energy level; providing foods that are easier to chew and swallow may be better choices for promoting intake. Foods that are protein and/or calorie dense include: - Whole milk and dairy products (eg, milkshakes), fruit smoothies - Granola, muffins, biscuits - Potatoes with sour cream and butter - Meat, fish, eggs, dried beans, almond butter - Pasta/rice dishes with cream sauce

The nurse is caring for a client with hepatic encephalopathy who is receiving newly prescribed lactulose. Follow-up would be required if the nurse: Hepatic encephalopathy is a complication of liver disease and is associated with elevated serum ammonia levels. Lactulose traps ammonia in the intestinal tract. Its laxative effect then expels the ammonia from the colon, resulting in decreased serum ammonia levels and correction of hepatic encephalopathy

3. holds the medication if the client has three soft stools a day Hepatic encephalopathy is a reversible neurologic complication of cirrhosis caused by increased ammonia levels in the blood. Lactulose reduces ammonia levels through fecal excretion and is the most common treatment for hepatic encephalopathy. The desired therapeutic effect of lactulose is the production of two or three soft bowel movements each day, and this therapeutic dose should be maintained until desired outcomes of therapy are reached and maintained (eg, improved mental status, decreased serum ammonia levels) (Option 3). Lactulose can be given orally on an empty stomach for faster results or mixed with water, juice, or milk to improve flavor.

The clinic nurse provides teaching for a client scheduled for a barium enema the next day. Which statement by the client shows a need for further instruction? This procedure is contraindicated for clients with acute diverticulitis as it may rupture inflamed diverticula and cause subsequent peritonitis.

4. "I will avoid laxatives after the procedure." Take a laxative (eg, magnesium hydroxide [Milk of Magnesia]) to assist in expelling the barium. Retained barium can lead to fecal impaction (Option 4). Before the procedure, clients undergo bowel preparation using cathartics (,magnesium citrate, polyethylene glycol) to empty stool from the colon and a clear liquid diet - avoid red and purple liquids, do not eat or drink anything 8 hours before the test, may experience abdominal cramping and an urge to poop . Post-op, expect the passage of chalky, white stool until all barium contrast has been expelled, drink plenty of fluids to promote hydration and eat a high-fiber diet to prevent constipation.. Laxatives after the procedure assist in expelling the contrast.

The nurse is reinforcing discharge teaching to the client. Which of the following client statements indicate that the teaching has been effective? Select all that apply.

4. "I will immediately report any dark stools to my health care provider. "5."I will request a prescription for varenicline from my health care provider." - To prevent new peptic ulcer formation or disease exacerbation, clients should avoid activities (eg, smoking) and drugs (eg, caffeine, alcohol, NSAIDs) that stimulate production of stomach acid and impair ulcer healing. Clients should be instructed to report signs and symptoms of a recurrence of gastrointestinal bleeding (eg, melena) and request a prescription for a smoking cessation aid (eg, varenicline).

The nurse assesses a client who has followed a vegan diet for several years. Which client statement indicates a potential nutritional deficiency?

4. "Sometimes my hands and feet get a tingling sensation." Clients who are vegan are at risk for deficiency of vitamin B12 (cobalamin), which is primarily supplied by animal products. Chronic vitamin B12 deficiency may precipitate megaloblastic anemia and neurological symptoms across the entire nervous system, from peripheral nerves to the spinal cord and brain. Manifestations of chronic deficiency include: Peripheral neuropathy (eg, tingling, numbness) (Option 4) Neuromuscular impairment (eg, gait problems, poor balance) Memory loss/dementia (in cases of severe/prolonged deficiencies)

A client is admitted with severe acute pancreatitis (LUQ). While obtaining the client's blood pressure, the nurse notices a carpal spasm. What laboratory result would the nurse assess in response to this symptom? Pancreatitis is an acute inflammation of the pancreas that results in autodigestion. The most common causes are cholelithiasis and alcoholism. Classic presentation includes severe epigastric pain radiating to the back due to the retroperitoneal location of the pancreas. The pancreatic enzymes (amylase and lipase) are elevated. Serious complications to monitor for include hyperglycemia, hypovolemia (capillary leak → third spacing), latent hypoxia or acute respiratory distress syndrome (ARDS), peritonitis, and hypocalcemia.

4. Hypocalcemia Pancreatitis can cause hypocalcemia, but the etiology is unclear. Chvostek's (facial twitching) and Trousseau's (carpal spasm) signs are an indication of hypocalcemia from the decrease in threshold for contraction. Sustained muscle contraction (tetany) and decreased cardiac contractility (cardiac arrhythmia) are concerns related to hypocalcemia.

An adolescent client seen in the ambulatory care center is going on a one-week fasting regimen of water and juice to jump start weight loss. The nurse's response is based on an understanding of which of the following?

4. Initial weight loss during fasting is primarily from fluid loss Fasting can cause multiple health problems, including increased stress, slowing of the body's metabolism, muscle damage, fluid loss, increased hunger, depletion of nutrients, and physical symptoms such as headache, dizziness, fatigue, and muscle weakness. Fasting = 1-2 days can lead to health problems, body will use protein, fat, and stored carbs as energy sources. Appetite will be suppressed with a few days but return when regular intake

The nurse is admitting a client with cholelithiasis and acute cholecystitis. Suddenly, the client vomits 250 mL of greenish-yellow stomach contents and reports severe pain in the right upper quadrant with radiation to the right shoulder. Which intervention would have the highest priority?

4. Maintain nothing-by-mouth (NPO) status - The highest priority intervention for an actively vomiting client with cholelithiasis is maintenance of strict NPO status to avoid additional gallbladder stimulation. Additional collaborative interventions (see table) for cholecystitis should also be taken into account. - Promethazine 25 mg suppository is the second priority. Promethazine promotes the relief of nausea and vomiting and minimizes further fluid loss. (Option 2) - Obtaining fluid and electrolyte replacement with sodium chloride 100 mL/hr is the third priority and assists in the maintenance of fluid balance. (Option 3) - Insertion of a nasogastric (NG) tube to low suction is the fourth priority. NG suction provides gastric decompression, alleviates nausea and vomiting, and promotes bowel rest.

The nurse is evaluating the effectiveness of parenteral nutrition for a client with severe oropharyngeal and esophageal mucositis who has been receiving total parenteral nutrition for the past 10 days. Which of the following findings would indicate that parenteral nutrition has been effective? Select all that apply. Click the exhibit button for additional information. Total parenteral nutrition (TPN), a form of parenteral nutrition administered into a central vein, is used when a client cannot absorb adequate nutrition from the digestive tract because of dysfunction or increased caloric needs due to illness (eg, burns, chemotherapy).

4. Prealbumin: 30 mg/dL (300 mg/L) 5. Weight: 116 lb (52.6 kg) -Clinical findings indicating TPN effectiveness include increased prealbumin that indicates a positive nitrogen balance and, because of its 2-day half-life, is an accurate indicator of improved nutritional status (Option 4); another indicator is weight gain or maintenance that does not result from fluid retention (Option 5). Administration of TPN increases the risk for metabolic imbalances (eg, hypokalemia, refeeding syndrome) and requires frequent serum electrolyte levels and adjustment of TPN components; however, this is not a measurement of therapeutic effectiveness.

Which prescription should the nurse question when caring for a hospitalized client diagnosed with acute diverticulitis? Acute care for diverticulitis focuses on allowing the colon to rest and the inflammation to resolve. This includes the following:

4. Prepare for barium enema in AM (Option 4) Any procedure or treatment that increases intraabdominal pressure (lifting, straining, coughing, bending), increases peristalsis (laxative, enema), or could lead to perforation or rupture of the inflamed diverticula should be avoided during the acute disease process. A barium enema may be used after treatment with antibiotics and the inflammation is resolved. Diagnostic examinations, such as abdominal x-rays or CT scans, may be used without risking rupture. - IV antibiotic therapy - to cover the gram-negative and anaerobic organisms that reside in the colon and contribute to diverticulitis; these commonly include metronidazole (Flagyl) plus trimethoprim/sulfamethoxazole (TMZ) (Bactrim or Bactrim DS; Septra) or ciprofloxacin (Cipro) (Option 1) - NPO status - more acute cases require complete rest of the bowel (NPO status); less severe cases may be handled at home, and clients may tolerate a low-fiber or clear liquid diet (Option 3) - NG suction - in severe cases of abdominal distention, nausea, or vomiting (Option 2) - IV fluids - prevent dehydration - Bed rest

The nurse is reinforcing education to a client with irritable bowel syndrome who is experiencing diarrhea. Which of these meals selected by the client indicates an understanding of diet management

4. Steak, tomato basil soup, and cornbread To manage IBS, clients should restrict gas-producing foods (eg, bananas, cabbage, onions, beans, broccoli, cabbage; bagels) caffeine/coffee; alcohol/wine; fermentable oligo-, di-, and monosaccharides and polyols (FODMAPs) (eg, honey, high-fructose corn syrup, wheat); and other gastrointestinal (GI) irritants (eg, spices, hot/cold food or drink, dairy products, fatty foods). Clients should gradually increase fiber intake (eg, whole grains, legumes, nuts, fruits, vegetables) as tolerated. Foods that are generally well tolerated include proteins, breads, and bland foods (Option 4).

A client comes to the clinic for a follow-up visit after a Billroth II surgery (gastrojejunostomy). The client reports occasional episodes of sweating, palpitations, and dizziness 30 minutes after eating. Which nursing action is most appropriate? Billroth II surgery (gastrojejunostomy) removes part of the stomach and shortens the upper gastrointestinal tract. After a partial gastrectomy, many clients experience dumping syndrome, which occurs when gastric contents empty too rapidly into the duodenum, causing a fluid shift into the small intestine. This results in hypotension, abdominal pain, nausea/vomiting, dizziness, generalized sweating, and tachycardia

4. Teach the client to lie down after eating To reduce the occurrence of symptoms/dumping syndrome, clients should avoid fluids with meals and lie down after eating to SLOW gastric emptying (Option 4). An upright or sitting position INCREASES the force of gravity, which INCREASES the rate of gastric emptying. Hypoglycemia can cause symptoms similar to those of dumping syndrome (eg, sweating, dizziness) but is unlikely to occur 30 minutes after eating. Clients should avoid consuming fluids with meals, which causes stomach contents to pass faster into the jejunum and worsens symptoms. Fluid intake should occur at least 30 minutes before/after meals.

The nurse is assessing a client who had an esophagogastroduodenoscopy 3 hours ago and is reporting increasing abdominal pain. Which of the following findings would require immediate follow-up?

4. Temperature of 101.6 F (38.7 C) An esophagogastroduodenoscopy (EGD) involves passing an endoscope down the esophagus to visualize the upper gastrointestinal structures (eg, esophagus, stomach, duodenum). Perforation of the gastrointestinal tract is a life-threatening complication of EGD that can lead to peritonitis and sepsis. Signs of perforation include increased temperature, increasing abdominal pain/tenderness, restlessness, tachycardia, and tachypnea. The nurse should notify the health care provider immediately if the client develops a fever (Option 4). - Some drowsiness is expected, and no intervention is necessary if the client is easily aroused. - It may take a few hours for the gag reflex to return. Absent gag reflex after a prolonged period (eg, 4 hr) should be reported to the health care provider. - A sore throat is expected after certain procedures (eg, EGD, intubation) due to local irritation. Warm saline gargles can provide some relief and can be offered to the client when the gag reflex has returned.

The nurse is caring for a client with acute pancreatitis admitted 2 days ago. Which assessment finding is most concerning?

4. Temperature of 102.2 F (39 C) with increasing abdominal pain Clients with acute pancreatitis are at risk for pancreatic abscess development. This mainly results from secondary infection of pancreatic pseudocysts or pancreatic necrosis. High fever, leukocytosis, and increasing abdominal pain may indicate abscess formation (Option 4). The abscess must be treated promptly to prevent sepsis. The health care provider should be notified immediately as antibiotic therapy and immediate surgical management may be required. Expected with pancreatitis = elevated blood glucose; severe burning midepigastric pain that radiates to the back (seek relief by positioning knees to chest), steatorrhea (fatty, yellow, smelling stools d/t increase lipid production)

The nurse is evaluating a client with liver cirrhosis who received IV albumin after a paracentesis to drain ascites. Which assessment finding indicates that the albumin has been effective? Ascites is the accumulation of fluid in the peritoneal space that often occurs in clients with liver cirrhosis. Ascitic fluid increases abdominal pressure, resulting in weight gain, abdominal distension and discomfort, and shortness of breath. Paracentesis (ie, needle insertion through the abdomen into the peritoneum to remove ascitic fluid) is often performed to reduce symptoms of ascites. However, clients undergoing paracentesis must be monitored closely for hypotension as changes in abdominal pressure often result in systemic vasodilation.

4. Vital signs remain within the client's normal parameters Clients may receive IV albumin (a colloid) after paracentesis, which increases intravascular oncotic pressure resulting in increased intravascular fluid volume (blood in patient's circulatory system). Albumin administration prevents hypotension and tachycardia by mitigating hemodynamic changes associated with paracentesis (Option 4). Asterixis (ie, flapping hand tremors during arm extension) occurs due to elevated blood ammonia levels. Lactulose is commonly used to treat asterixis as it promotes ammonia excretion Decreased abdominal circumference and improved respiratory effort occur in clients with ascites after ascitic fluid is removed via paracentesis. Albumin does not directly reduce ascitic fluid volume.

For each finding below, click to specify if the finding is consistent with the disease process of acute pancreatitis, acute cholecystitis, and bleeding gastric ulcer. Each finding may support more than one disease process.

Acute Cholecystitis: Epigastric abdominal pain, Increased abdominal pain after eating - Inflammation of the gallbladder may occur due to gallstones or cholestasis. Severe pain occurs after eating due to impaired or blocked release of bile from the common bile duct. Pain is often located in the epigastric area of the upper abdomen or right upper quadrant. (RUQ) Acute Pancreatitis: Chronic alcohol use, Epigastric abdominal pain, Pain relief when leaning forward, Increased abdominal pain after eating - Acute pancreatitis. Inflammation of the pancreas is most often caused by gallstones or chronic alcohol consumption. Severe pain occurs after eating due to release of pancreatic enzymes in and around the pancreas, causing inflammation and autodigestion of pancreatic tissue. Pain is usually located in the epigastric area and/or left upper abdomen (LUQ) and may be partially relieved by leaning forward, which decreases abdominal tension. Bleeding Gastric Ulcer: Chronic alcohol use, Epigastric abdominal pain, Increased abdominal pain after eating - Bleeding gastric ulcer. Chronic alcohol consumption weakens the gastrointestinal lining and may lead to gastric ulcer development. Severe pain occurs after eating due to increased gastric acid secretion, which inflames ulcerated tissue. Pain is located in the epigastric area of the upper abdomen. Dark stool may be present as the bowels digest blood from the ulcer.

For each finding below, click to specify if the finding is consistent with the disease process of acute cholecystitis or acute myocardial infarction.

Acute Cholecystitis: diaphoresis (sweating), nausea and vomiting, epigastric abdominal pain, symptom onset after meals, abdominal tenderness on deep inspiration,. - Acute cholecystitis: Gallstones or other mechanical obstruction may lead to inflammation, ischemia, and infection of the gallbladder. Pain may be vague or located in the epigastric area or RUQ of the abdomen, radiate to the right shoulder or scapula, and cause tenderness on deep inspiration (ie, positive Murphy sign). Pain and nausea may begin after ingestion of a high-fat or large meal, which induces bile secretion by the gallbladder. Fever, tachycardia, and diaphoresis may occur as a result of the inflammatory response. Acute myocardial infarction: diaphoresis (sweating), nausea and vomiting, abdominal/epigastric pain, symptom onset after meals - Ischemia and infarction of the heart muscle causes substernal chest pain or pressure; in addition, referred pain may be present in epigastric, abdominal, shoulder, arm, jaw, or back. Epigastric pain is particularly common with inferior wall MI as the heart rests on the diaphragm. The pain may occur at rest or following exertion, emotional stress, or meals. Common associated symptoms include nausea, vomiting, diaphoresis, and shortness of breath. Although epigastric and RUQ pain may be present, significant tenderness on examination is not present because there is no underlying inflammation in those areas.

Acute Upper GI Bleed vs. Acute Lower GI Bleed

Acute Upper GI Bleed: NSAID use (risk factor of Helicobacter pylori infection), melena (Dark, tarry stool) hematemesis (vomiting blood), history of Helicobacter pylori infection (risk factor for peptic ulcer formation) Acute Lower GI Bleed: History of diverticulosis: hollow, outpouchings from the intestine, develop, usually in the large and, occasionally, small intestine.

The health care provider suspects the client is experiencing upper gastrointestinal bleeding. For each potential prescription, click to specify whether the prescription is anticipated or not anticipated for the care of the client.

Anticipated: Place the client on NPO status (reduce the risk of continued bleeding and vomiting --> initiate prior to esophagogastroduodenoscopy to reduce aspiration risk), administer isotonic IV fluid bolus (restore circulating fluid volume and maintain perfusion of vital organs.), administer proton pump inhibitor IV (-azole i.e. pantoprazole) to reduce gastric acid secretion and prevent further irritation and breakdown of suspected peptic ulcers), collect blood samples for type and crossmatch (ensure blood type compatibility before initiating a blood transfusion. This client's hemoglobin and hematocrit levels are low, and the client continues to have active bleeding. Therefore, a blood transfusion should be anticipated to increase blood volume and improve oxygenation and perfusion) Not anticipated: Administer heparin for deep venous thrombosis prophylaxis (Anticoagulation will prolong bleeding and increase risk for hemorrhagic shock. Anticoagulation is CONTRAINDICATED for clients with active GI bleeding.

CT Angiography

CT angiography is a type of CT scan that uses IV contrast to visualize blood vessels

For each finding below, click to specify if the finding is consistent with the disease process of celiac disease or irritable bowel syndrome. Each finding may support more than one disease process Celiac disease is an autoimmune disorder in which clients cannot process gluten, a protein in many grains (eg, barley, rye, wheat). Irritable bowel syndrome is a functional disorder, characterized by chronic abdominal pain associated with altered bowel habits (eg, diarrhea and/or constipation). Pain is often relieved with bowel movements or the passage of gas. Symptoms of celiac disease include diarrhea, pale-colored stools, iron deficiency anemia, nutrient malabsorption, and weight loss. Symptoms of irritable bowel syndrome include diarrhea and/or constipation.

Celiac Disease: diarrhea, pale-colored stools, iron deficiency anemia, nutrient absorption - s/s: diarrhea (often pale-colored stool) and malnutrition, weight loss. Stools are bulky and oily (floating) but do NOT contain blood or mucus. Nutrient malabsorption can also lead to specific micronutrient deficiencies (eg, iron deficiency anemia). Irritable Bowel Syndrome: diarrhea/constipation Other common symptoms include abdominal bloating, belching, flatulence, or mucus discharge with stooling. Because there is no inflammation, stools do not contain blood. In addition, weight loss, nutrient malabsorption, pale-colored stools, and micronutrient deficiencies (eg iron deficiency anemia) are not common.

The nurse should prepare the client for emergency ________ to treat the source of bleeding.

Esophagogastroduodenoscopy (EGD). - An EGD involves passing an endoscope down the esophagus to visualize the upper gastrointestinal (GI) structures (eg, esophagus, stomach, duodenum), identify the source of the bleed, and perform interventions to stop bleeding (eg, hemostatic clipping). To prepare the client for EGD, the nurse should ensure NPO status has been initiated to reduce the risk of aspiration. Interventions to stabilize the client (eg, IV fluids, blood transfusion) are often initiated before EGD

The nurse is providing teaching about gluten-free foods to the client. Drag the food item that indicates correct understanding of the teaching to the box on the right.

Gluten free food: - avocado - cheese - brown rice - whole milk - black beans - baked potatoes - corn tortilla Examples of gluten-free foods include: - Dairy (eg, cheese, whole milk) - Fats and oils (eg, avocados, butter) - Grains (eg, brown rice) - Protein (eg, black beans, meats, poultry) - Fruits and vegetables (eg, bananas, bell peppers, potatoes) NO: soy sauce, flour tortilla (breads, rolls, bagels - common source of gluten

The client is diagnosed with acute cholecystitis. For each prescription, click to specify if the prescription is indicated or not indicated for the care of the client.

Indicated: Administer IV antibiotics, Administer IV ondansetron, Initiate continuous IV fluids Not indicated: Encourage oral fluid intake, Initiate a low-fat diet - Encouraging oral fluid intake and a low-fat diet are not indicated in clients with acute cholecystitis. Fluid and electrolytes should be replaced intravenously, and the client should be kept strictly NPO in the preoperative period to avoid stimulating the gallbladder to secrete bile and to contract. Following a cholecystectomy, the client should follow a low-fat diet.

The client is diagnosed with acute pancreatitis. For each potential prescription, click to specify if the prescription is indicated or contraindicated for the care of the client.

Indicated: Administer antiemetics IV, Administer pain medication IV, Administer continuous IV fluids,Check blood glucose every 6 hours, Position the client with knees flexed toward the chest Not indicated: Administer pancreatic enzymes PO - NPO status is maintained to inhibit release of pancreatic enzymes. Oral medications are contraindicated until symptoms are resolved and diet is resumed/tolerated. Pancreatic enzymes are usually needed for clients with chronic pancreatitis because a large part of the pancreas is damaged.

The client has returned to the clinic to review diagnostic results. For each potential intervention, click to specify if the intervention is indicated or not indicated for the care of the client at this time

Indicated: Teach the client about a gluten-free diet, Instruct the client to avoid alcoholic beverages made with grains - Teaching the client about a gluten-free diet. Eliminating gluten from the diet reduces the risk of nutritional deficiencies. Eating even small amounts of gluten will damage the intestinal villi although the client may have no clinical symptoms. - Instructing the client to avoid alcoholic beverages made with grain (eg, beer, malted beverages) because they are a source of gluten. Not indicated: Administer antibiotics, Administer immunosuppressants -Celiac disease is NOT INFECTIOUS; therefore, administering antibiotics is not indicated. Celiac disease is treated with diet modification (ie, gluten-free diet). Administering immunosuppressants is not indicated because the disease can be easily managed by adhering to a gluten-free diet. Immunosuppressants are associated with risk of infection and malignancy.

The nurse assesses a client with suspected acute pancreatitis and anticipates the client reporting pain in which anatomical area?

Left upper quadrant radiating to the back The client with acute pancreatitis will report a sudden onset of unrelenting, severe pain in the left upper quadrant (LUQ) or midepigastric area of the abdomen that often radiates to the back. The pain is referred to the back as the pancreas is a retroperitoneal organ. Pain improves with leaning forward and worsens with lying flat. Kidney stones = Left flank radiating to the left groin area Appendicitis = as periumbilical pain progressing to the right lower quadrant (RLQ) Cholecystitis (inflammation of the gallbladder ) = pain in the right upper quadrant (RUQ)

The client is most likely experiencing gastrointestinal bleeding related to

Peptic Ulcer Disease (upper GI bleeding) - PUD is one of the most common causes of upper GI bleeding due to erosion and ulceration of the protective layers (ie, mucosa) of the upper GI tract (eg, esophagus, stomach, duodenum). The client has multiple risk factors for PUD, including history of Helicobacter pylori infection, chronic NSAID use, smoking, and daily alcohol consumption.

The client reports nausea, vomiting, and abdominal pain that has gradually worsened over the past 8 hours. The pain began as vague periumbilical discomfort in the morning that became more severe, sharp, and localized to the right lower abdominal quadrant.The last bowel movement was 2 days ago and was liquid stool. The right lower quadrant of the abdomen is tender. Appendicitis is inflammation of the appendix commonly caused by obstruction of the appendiceal lumen (eg, hard fecal matter). Luminal distension, mucous collection, and vascular compression/thrombosis within the appendix lead to tissue ischemia, necrosis, and infection. Characteristic abdominal pain begins in the umbilical area and then migrates to the right lower quadrant (ie, McBurney point). Guarding and rebound tenderness are common due to surrounding peritoneal irritation. The body's inflammatory response causes leukocytosis and fever. Distension and pressure within the intestine induce nausea, vomiting, and anorexia.

Potential Condition: Appendicitis (RLQ) Actions to Take: Initiate NPO status, Prepare the client for surgery Parameters to Monitor: Abdominal Rigidity, Acute absence of pain The nurse should prepare the client for surgery (ie, appendectomy) and initiate NPO status to decrease risk of aspiration during anesthesia. Increased pressure and inflammation within the appendix can lead to perforation (ie, rupture), which can cause life-threatening peritonitis and sepsis. Signs of perforation include abdominal rigidity and acute absence of pain that occurs with a sudden reduction of intraluminal pressure. Manifestations of gastroenteritis (ie, inflammation of the gastric and intestinal mucosa) include nausea, vomiting, diarrhea, and abdominal pain. The nurse should monitor stool culture results to determine the causative agent. Manifestations of nephrolithiasis (ie, kidney stones) are related to obstruction of the urinary tract and include flank pain, hematuria, and dysuria. Although some kidney stones are small enough to pass without intervention, large stones commonly require lithotripsy. Urine sediment indicates passing of the stones. A bowel obstruction causes abdominal distension, colicky abdominal pain, bilious vomiting, and inability to pass stool. Treatment includes insertion of a nasogastric tube for gastric decompression, as well as monitoring gastric fluid color for signs of complications (e

Purpose of JP drain

Prevent fluid buildup in a closed space, which can put tension on the suture line and compromise the integrity of the incision, increase the risk for infection, and decrease wound healing. To drain excess fluid from a wound bed to prevent a hematoma or seroma

Dumping syndrome

Rapid emptying of gastric contents into small intestines (duodenum). Client experience ab pain/cramping, nausea, vomiting, explosive diarrhea, weakness, dizziness, palpitations & tachycardia. ONLY clients who experience dumping syndrome should lay supine for a short period after eating.

Low Fowler's Position with knees bent

Supine with HOB elevated 15-30 degrees

A client with hematemesis and black stools is most likely experiencing an acute gastrointestinal (GI) bleed. GI bleeding is a life-threatening condition that can lead to hemorrhagic shock without immediate intervention. Hematemesis (vomiting blood) + black stools = GI bleed

The nurse should immediately follow up on: - Chronic NSAID use (eg, ibuprofen), a common cause of drug-induced upper GI bleeds that must be discontinued. - Hypotension and tachycardia, manifestations of hypovolemia. Hypotension occurs with decreased cardiac output, and tachycardia is a compensatory mechanism to promote maximum perfusion to vital organs. Because this client has a history of hypertension, even borderline low blood pressure is considered abnormal. - Syncope (ie, passing out), a clinical finding associated with hypovolemia caused by decreased perfusion to the brain. - Hematemesis (ie, vomiting blood), indicative of bleeding in the upper GI tract (eg, stomach ulcers, esophageal varices). - Dark/black and/or tarry stools, most often associated with upper GI bleeding; blood becomes partially digested as it passes through the GI tract, resulting in the dark color. Tobacco and alcohol, not weed = common risk factors for upper GI bleeding. Crepitus, caused by the rubbing of two rough joint surfaces, = consistent with chronic degenerative joint disease (ie, osteoarthritis) S/s of hypovolemia = hypotension, tachycardia, & syncope)

ulcerative colitis (UC) = LOWER GI BLEEDING

Ulcerative colitis (UC) is a chronic inflammatory bowel disease characterized by inflammation and ulceration of the large intestine (colon) that results in abdominal pain, frequent bouts of bloody diarrhea, anorexia, and anemia. (clients have numerous bowel movements (ie, ≥6/day), which can contribute to significant blood loss through the stool.)

Small Bowel Obstruction (SBO) For each potential intervention, click to specify if the intervention is indicated or not indicated for the care of the client. Clients with SBO are at risk for fluid, electrolyte, and nutritional imbalances due to decreased intestinal absorption. Clients may develop bowel necrosis and perforation due to impaired intestinal blood flow --> peritonitis and sepsis.

physical blockage of normal passage of intestinal contents in the small intestine --> colicky abdominal pain, vomiting,abdominal distension, bilious vomiting, and inability to pass flatus or stool. Indicated: Administer IV antiemetic, Insert a nasogastric tube, Obtain an abdominal CT scan - Inserting a nasogastric tube for gastrointestinal decompression to reduce abdominal distension and improve intestinal blood flow - Administering IV antiemetics to prevent further fluid and electrolyte imbalance from vomiting - Preparing the client for abdominal CT scan to determine the size and location of intestinal obstruction - Administering IV fluids to improve fluid volume status Not Indicated: place client on soft diet, administer a stimulant laxative -In clients with SBO, bowel rest (ie, NPO status) with gastric decompression is prescribed; therefore, a soft diet is not indicated. - Stimulant laxatives increase intestinal motility and are not indicated for clients with intestinal obstruction due to the risk for bowel perforation.


संबंधित स्टडी सेट्स

Recursive Formulas for Arithmetic Sequences

View Set

SUBJECT and OBJECT PRONOUNS: Replace the object and subject with the correct pronoun!

View Set

Chapter 5: Traffic-Control Devices

View Set

BJU Physical Science (6th ed.) - Chapter 4

View Set